-
Notifications
You must be signed in to change notification settings - Fork 0
/
Aufgabe2.tex
81 lines (69 loc) · 3.88 KB
/
Aufgabe2.tex
1
2
3
4
5
6
7
8
9
10
11
12
13
14
15
16
17
18
19
20
21
22
23
24
25
26
27
28
29
30
31
32
33
34
35
36
37
38
39
40
41
42
43
44
45
46
47
48
49
50
51
52
53
54
55
56
57
58
59
60
61
62
63
64
65
66
67
68
69
70
71
72
73
74
75
76
77
78
79
80
81
\documentclass[crop=false]{standalone}
\usepackage[utf8]{inputenc}
\usepackage{amsmath}
\usepackage[dvipsnames]{xcolor}
\usepackage{pdfpages}
\usepackage{enumerate}
\usepackage{amssymb}
\usepackage[framemethod=default]{mdframed}
\usepackage[nomarginpar,left=2cm,right=2cm,top = 2cm, bottom = 2cm]{geometry}
\renewcommand{\thesubsection}{\thesection.\alph{subsection}}
\renewcommand{\thesubsubsection}{\thesection.\alph{subsection}.\roman{subsubsection}}
\mdfdefinestyle{theoremstyle}{%
linecolor=black,linewidth=.3pt,%
frametitlerule=true,%
frametitlebackgroundcolor=blue!5,
innertopmargin=\topskip,nobreak=true,
}
\mdfdefinestyle{style2}{frametitle={},%
linewidth=.3pt,topline=true,backgroundcolor=blue!3!green!8!}
\mdtheorem[style=theoremstyle]{task}{Angabe}
\newmdenv[style = style2,title=false]{solution}
\begin{document}
\begin{task}[Stabilität und Linearisierung]
Ein Ball liege auf einem Rad. Das Rad werde durch ein Drehmoment $M$ angetrieben.
Durch die Drehung des Rades soll der Ball in der oberen Ruhelage balanciert werden.
Die interessierende Ausgangsgröße sei der Winkel $\alpha$, um
den der Ball von der Mitte abweicht. Mit nicht physikalisch motivierten Parametern erhält man die Differentialgleichungen zweiter Ordnung
\[
\begin{aligned} 3 \dot{\omega}-2 \ddot{\alpha} &=M \\-2 \dot{\omega}+3 \ddot{\alpha}-\frac{11}{2} \sin (\alpha) &=0 \end{aligned}
\]
mit Drehmoment $M$ und Winkelgeschwindigkeit $\omega$. Der
Winkel des Rades wird nicht modelliert.
\emph{HINWEIS: Nebenstehende Skizze dient nur der Veranschaulichunq. Die Bewegunqsgleichungen sind gegeben.}
\begin{enumerate}[i]
\item Bringen Sie das nichtlineare System auf Zustandsdarstellung
\[
\begin{aligned} \dot{\mathbf{x}} &=\mathbf{f}(\mathbf{x}, u) \\ y &=g(\mathbf{x}, u) \end{aligned}
\]
\begin{solution}
Da $\alpha$ der Ausgang des Systems sein soll, muss es jedenfalls im Zustandsvektor $\mathbf{x}$ vorkommen.
Es müssen die beiden gegebenen Gleichungen nach $\ddot{\alpha}$ und $\dot{\omega}$ aufgelöst werden.
\[\mathbf{x} = \begin{pmatrix} \alpha \\ \dot{\alpha} \\ \omega \end{pmatrix}, \quad
\dot{\mathbf{x}} = \begin{pmatrix} \dot{\alpha} \\ \ddot{\alpha} \\ \dot{\omega} \end{pmatrix} =
\begin{pmatrix} \dot{\alpha} \\ \frac{33}{10}\sin{\alpha} + \frac{2}{5} M\\ \frac{11}{5} \sin{\alpha} + \frac{3}{5} M \end{pmatrix}
\]
\end{solution}
\item Berechnen Sie die stationäre Stellgröbe $u_{s}$ so, dass sich eine Ruhelage $\mathbf{x}_{s}$ mit
$\alpha_{s}=0$ einstellt und geben Sie die Ruhelage $\mathbf{x}_{s}$ an.
\begin{solution}
\[ \mathbf{x}_s = \begin{pmatrix} \alpha_s \\ \dot{\alpha_s} \\ \omega \end{pmatrix} = \begin{pmatrix} 0 \\ 0 \\ 0 \end{pmatrix}, \quad
\dot{\mathbf{x}_s} = \begin{pmatrix} 0 \\ 0 \\ 0 \end{pmatrix} =
\begin{pmatrix} 0 \\ \frac{33}{10}\sin{0} + \frac{2}{5} M\\ \frac{11}{5} \sin{0} + \frac{3}{5} M \end{pmatrix} \rightarrow M = 0\]
\end{solution}
\item Linearisieren Sie das System um die zuvor berechnete Ruhelage und geben Sie es
in Zustandsdarstellung an:
\[
\begin{aligned} \Delta \dot{\mathbf{x}} &=\mathbf{A} \Delta \mathbf{x}+\mathbf{b} \Delta u \\ \Delta y &=\mathbf{c}^{T} \Delta \mathbf{x} \end{aligned}
\]
\begin{solution}
\[\mathbf{A} = \begin{pmatrix} 0 & 1 & 0 \\ \frac{33}{10} & 0 & 0 \\ \frac{11}{5} & 0 & 0\end{pmatrix}, \quad \mathbf{b} = \begin{pmatrix}0 \\ \frac{2}{5} \\ \frac{3}{5}\end{pmatrix}, \quad \mathbf{c}^T = \begin{pmatrix}1 &0 &0\end{pmatrix}\]
\end{solution}
\item Beurteilen Sie die Stabilität der Ruhelage $\mathbf{x}_{s}$ von Punkt (ii) anhand des linearisierten Systems.
\begin{solution}
\[\text{Det}\left( \mathbf{A} - \mathbf{E}\lambda \right) = - \lambda^3 + \lambda \frac{33}{10} \rightarrow \lambda_1 = 0, \quad \lambda_{2,3} = \pm \sqrt{\frac{33}{10}}\]
Es existiert mindestens ein Eigenwert mit positiven Realteil, somit ist das System instabil.
\end{solution}
\end{enumerate}
\end{task}
\end{document}